Получение фотонного пропагатора

В книге Пескина и Шредера на странице 297 при выводе фотонного пропагатора авторы говорят, что

(9.57б) ( к 2 г мю ν + ( 1 1 ξ ) к мю к ν ) Д Ф ν р ( к ) "=" я дельта мю р

С решением, приведенным в следующей строке уравнения (9.58), как

(9,58) Д Ф мю ν ( к ) "=" я к 2 + я ϵ ( г мю ν ( 1 ξ ) к мю к ν к 2 )

Который является распространителем. Я могу проверить это уравнение, вставив Д Ф мю ν ( к ) в первое уравнение, но я понятия не имею, как на самом деле решить Д Ф ν р ( к ) от ( 9,57 б ) . Если кто-то может помочь, это было бы очень признательно.

Ответы (2)

Д мю ν "=" А г мю ν + Б к мю к ν с A и B двумя неизвестными функциями скаляра k ^ 2. Два тензора после A и B являются единственными возможными лоренц-инвариантными тензорами. Просто подключайте и вычисляйте неизвестные функции.

Итак, в общем, я делаю анзац всех возможных лоренц-инвариантных членов, которые появляются в матрице, которую я хочу инвертировать?
Да это оно.

Это просто тензорное уравнение гласит: А мю ν Д ν р "=" я дельта мю р , где А мю ν "=" к 2 г мю ν + ( 1 1 ξ ) к мю к ν . Что нам нужно сделать, чтобы найти его Обратное А мю ν . Конечно, вы можете найти его методом грубой силы методами линейной алгебры, но ответ угадать проще.